Please confirm topic selection

Are you sure you want to trigger topic in your Anconeus AI algorithm?

Please confirm action

You are done for today with this topic.

Would you like to start learning session with this topic items scheduled for future?

Review Question - QID 108732

In scope icon M 6 E
QID 108732 (Type "108732" in App Search)
A 29-year-old woman presents to the emergency department with a 1-day history of fever. She says that it started spontaneously and that it hasn't gotten any better with either rest or acetaminophen. Her past medical history is significant for homelessness, unprotected sexual intercourse with multiple partners, and substance use disorder with multiple substances. Her temperature is 103.5°F (39.5°C), blood pressure is 100/70 mmHg, pulse is 112/min, respirations are 18/min, and oxygen saturation is 93% on room air. On physical exam, a murmur is heard along the left mid-sternal border. The pulmonary exam reveals minor bibasilar crackles. The patient's upper limbs demonstrate many bruises and scars in the antecubital fossa. The patient is started on vancomycin and gentamicin and sent for echocardiography. Based on the results of the echocardiogram, the patient undergoes surgery to remove multiple vegetations from the tricuspid valve. Vancomycin and gentamicin are continued over the next 5 days. On postoperative day 5, the patient presents with bleeding from her gums, oozing from her surgical sites, and recurrent epistaxis. Lab values are obtained as seen below:

Serum:
Na+: 135 mEq/L
Cl-: 90 mEq/L
K+: 4.4 mEq/L
HCO3-: 23 mEq/L
BUN: 20 mg/dL
Glucose: 110 mg/dL
Creatinine: 1.0 mg/dL
Ca2+: 10.1 mg/dL
AST: 9 U/L
ALT: 9 U/L

Leukocyte count and differential:
Leukocyte count: 6,000 cells/mm^3
Lymphocytes: 20%
Monocytes: 1%
Neutrophils: 78%
Eosinophils: 1%
Basophils: 0%
PT: 27 seconds
aPTT: 84 seconds
D-dimer: < 50 µg/L

Hemoglobin: 14 g/dL
Hematocrit: 40%
Platelet count: 150,000/mm^3
Mean corpuscular volume: 110 fL
Mean corpuscular hemoglobin concentration: 34 g/dL
RDW: 14%

Which of the following is the most likely cause of this patient's current symptoms?